SOLUTIONNAIRE: SÉANCE 1

publicité
1-803 Analyse Microéconomique
Solutions aux Exercices
Éléments de solution aux questions et exercices
du livre de Pindyck & Rubinfeld
Chapitre 1
Question 4
Une analyse économique positive décrit ce qui est. Une analyse économique normative décrit
ce qui devrait être. Ici, a) positif b)positif
Chapitre 2
Question 1
Étant donné que la courbe d'offre de court terme est stationnaire, ce déplacement de la courbe
de demande cause une demande excédentaire à court terme, ce qui requiert une certaine forme
de rationnement. Alors qu'une forme de rationnement serait de faire attendre les
consommateurs en file (possiblement au soleil!), une alternative serait la hausse des prix
(surenchère) par les consommateurs prêts à payer plus cher ce bien. Le prix augmente jusqu'à
ce que la quantité demandée et la quantité offerte soient égales.
Question 2
A
B
P
P
O1
O
O0
P1
P1
P0
P0
D1
D
D0
Q0
Q1
Q1
Q
238
Q0
Q
1-803 Analyse Microéconomique
Solutions aux Exercices
C
P
O
P0
P1
D0
D1
Q
Q1
Q0
Question 8
C
A et B
P
P
O1
O
O0
D1
D
D0
Q
239
Q
1-803 Analyse Microéconomique
Solutions aux Exercices
D et E
P
O0
O1
D
Q
Question 3
Élasticité-prix de demande = Ep =  %Q donc Ep = -6% = -2
 %P
3%
Question 4
Les consommateurs prennent habituellement le temps d'agir face aux variations dans les prix.
Certains changements sont rapides, d'autres moins. Alors les différences entre les élasticités
de court et de long terme dépendent de la vitesse de réaction des consommateurs face aux
changements de prix.
La demande pour les biens non-durables change probablement très peu à court terme sauf si
les gens sont capables de modifier rapidement leurs habitudes de consommation. D'un autre
côté, la quantité demandée pour les biens durables (ex: t.v.) peut changer drastiquement lors
d'une variation de prix. L'influence première de ce changement serait de retarder l'achat de
ces biens mais éventuellement, les consommateurs devront les changer s'ils deviennent
désuets ou défectueux.
240
1-803 Analyse Microéconomique
Solutions aux Exercices
Exercice 1
a) Ep= (P/ Qd)(Qd/P). Ici, une hausse du prix de 20$ cause une baisse de quantité
demandée de 2 millions. Donc, Q/P = -2/20 = -0.1
Si P=80 et Q=20, Ep= (80/20)* -0.1 = -0.4
Si p=100 et Q=18, Ep= (100/18)* -0.1 = -0.56
b) Élasticité-prix de l'offre = Ep = P
Qs
 Qs
P
Ici, une hausse de prix de 20$ cause une hausse de quantité offerte de 2 millions. Donc,
Qs/P = 2/20 =0.1
Si P=80 et Q=16, Ep= (80/16)* 0.1 = 0.5
Si P=100 et Q=18, Ep= (100/18)* 0.1 = 0.56
c) L'équilibre se retrouve quand demande=offre. Ici, les quantités s'égalent à 18 et le prix
d'équilibre est de 100$.
Exercice 5
a) On sait, que pour une équation de demande linéaire, Ep= - b * (P /Q ) où -b est la pente de
la demande.
Ici Ep=-0.4, P =0.75 et Q =7.5 alors 0.4=-b*(0.75/7.5)  b=4
En substituant b,Q et P dans la demande, 7.5= a - 4*0.75  a=10.5
La demande est Qd= 10.5 - 4*P.
b) Cette baisse modifiera la demande comme suit: Qd = 0.8 (10.5 - 4P)= 8.4 - 3.2P
En égalant Qd = Qs, on trouve le nouvel équilibre:
8.4-3.2P=-4.5+16P  P =0.672 et Q =6.2496
241
1-803 Analyse Microéconomique
Solutions aux Exercices
Chapitre 3
Question 5
Si le TMS entre les 2 biens n'est pas égal au ratio des prix, le consommateur pourrait échanger
1 bien pour l'autre aux prix du marché pour obtenir un niveau supérieur de satisfaction.
Exercice 5
Voir réponse dans le manuel.
Exercice 10
a)
Soient A: aliments, V: vêtements, U(A,V) = A.V
V
12
6
4
U = 24
2
U = 12
A
6
12
b)
Prix de A:P(A)=1$/unité, Prix de V:P(V)=3$/unité, Budget Y=12$. Sa contrainte de
budget Y=P(A).A+P(V).V  12=A+3V ou V=4-1/3A. (Voir graphique en a)
c)
Le niveau d'utilité le plus élevé est présent lorsque la contrainte de budget est tangente
à la courbe d'indifférence la plus élevée. Sur le graphique, ce point est où A=6 et
V=2.
d)
Le TMS = -(pente de la courbe d'indifférence TMS=1/3, i.e. Jane est prête à
sacrifier 1/3 d'unité de V pour obtenir 1 unité de A.
e)
Si elle achète (3A, 3V), elle dépense tout son budget. Mais, son niveau d'utilité n'est
pas optimal car U=9. Le TMS serait égal à la pente de la courbe d'indifférence où elle
intersecte la droite de budget. À ce point, TMS < 1/3. Donc, elle sera prête à
échanger V pour A jusqu'à ce que son TMS = rapport des prix.
242
1-803 Analyse Microéconomique
Solutions aux Exercices
Chapitre 4
Question 1
La courbe de demande du marché est la somme horizontale des courbes de demandes
individuelles. La demande du marché démontre la relation entre chaque prix et la somme des
quantités individuelles. L'effet d'un changement de prix sur la quantité d'un consommateur
est moins élastique que celui sur le marché entier.
Question 2
Lorsque deux marques de commerce sont très similaires, un faible variation de prix d’une des
marques entraînera plusieurs consommateurs à se tourner vers une autre marque. Ce n’est
pas le cas lorsque le prix de l’ensemble des marques augmente. Donc, la demande pour les
skis de marque Head est plus élastique que la demande pour les skis de fond.
Question 3
Lorsque la demande excède l’offre, les consommateurs sont prêts à payer plus chers pour le
bien et ce, jusqu’à que la quantité demandée soit égale à la quantité offerte. Par conséquent,
si les consommateurs sont prêts à payer plus que 10$ pour maximiser leur utilité, l’utilité
marginale d’un billet supplémentaire (valeur) est supérieure à 10.
Question 5
a)
Un cours de mathématique et un cours d'économie, qui n'impliquent pas de conflits
d'horaire, peuvent être complémentaires ou substituts. S'ils impliquent un conflit
d'horaire (donnés en même temps), ils sont nécessairement substituts.
b)
Une balle et une raquette de tennis sont complémentaires.
c)
Le steak et le homard peuvent être substituts, mais ils sont complémentaires lorsque
servis ensemble.
d)
Deux modes de transport entre deux points identiques sont substituts l'un à l'autre. e)
voir c).
Question 6
Dans les deux premiers cas, on observe un déplacement de la courbe de demande. Par contre,
la baisse des coûts va entraîner un déplacement de la courbe d'offre et, du côté de la demande,
le nouveau prix et la nouvelle quantité d'équilibre vont être obtenus par un déplacement le
long de la courbe.
243
1-803 Analyse Microéconomique
Solutions aux Exercices
Question 9
On pose comme hypothèse que les trois groupes ont un accès égal à toutes les organisations
en concurrence avec l'Association of Business Economists. Le prix de la carte de membre
correspond à une part plus élevée du budget d'un étudiant que d'un gestionnaire junior, et d'un
gestionnaire junior par rapport à un sénior. Par conséquent, la demande d'un étudiant sera
plus élastique que celle d'un gestionnaire junior qui, à son tour, sera plus élastique que celle
d'un sénior.
Exercice 1
a)
Ep = %Q/%P. Pour les puces d'ordinateur Ep = -2, donc une hausse de 10% des
prix va entraîner une baisse de 20% des ventes. Dans le cas des lecteurs de disques,
Ep = -1, les ventes vont alors diminuer de 10%.
On pose TR1 = P1 Q1 , le revenu avant le changement de prix et TR2 = P2 Q2 , le revenu
après le changement de prix. Alors, la variation dans le revenu est TR = TR2 - TR1 .
Pour les puces d'ordinateur,
TR = P2 Q2 - P1 Q1 = (1,1 P1 ) (0,8 Q1 ) - P1 Q1 = -12%TR.
Pour les lecteurs de disques,
TR = P2 Q2 - P1 Q1 = (1,1 P1 ) (0,9 Q1 ) - P1 Q1 = -1%TR.
Donc, les revenus générés par les puces d'ordinateur diminuent alors que ceux générés
par les lecteurs de disque ne varient presque pas.
b)
On doit connaître les quantités et les prix des produits pour trouver les revenus de
vente totaux.
Exercice 6 (2e édition) - exercice 8 (3e édition)
Les livres et le café sont des biens normaux parce que leur consommation augmente
(diminue) lorsque le revenu de Bill augmente (diminue).
En a), lorsque son revenu augmente, sa consommation de livre augmente. Les livres sont des
biens normaux.
En b), il consomme plus de café lorsqu’il vend son livre. Le café est un bien normal.
En c), la chute de son revenu fait diminuer sa consommation de café et sa consommation de
livre. Les deux biens sont normaux.
En d), ses goûts changent complètement. Nous ne savons pas si c’est parce qu’il a plus
d’argent ou pour une autre raison..
Chapitre 5
244
1-803 Analyse Microéconomique
Solutions aux Exercices
Question 1
Une personne ayant de l'aversion au risque préfère moins de risque à une situation présentant
plus de risque. Pour d'autres, c'est le contraire. Ceci dépend des variations dans la fonction
d'utilité pour la richesse.
Question 3
L'espérance de l'utilité est la somme des utilités associées à chacun des résultats possibles
pondérée par la probabilité que chacune des possibilités se réalisent. Pour maximiser cette
espérance d'utilité, l'individu devra choisir l'option donnant la plus haute utilité moyenne.
Ceci requiert que le consommateur connaisse les probabilités de chacune de ces possibilités,
ce qui est difficile à évaluer dans bien des cas.
Question 7
De la même façon qu'une compagnie d'assurance peut réduire son risque par la diversification
en assurant un grand nombre individus non-apparentés, l'investisseur peut faire de même en
investissant dans un ensemble d'investissements indépendants comme un fond mutuel.
Puisque les variances de chacune des actions indépendantes sont elles aussi indépendantes, la
variance de ce portefeuille sera plus faible que si les éléments de celui-ci étaient détenus
séparément. À mesure que le nombre d'actions diverses augmente, la variance du taux de
rendement de ce portefeuille diminuera dans son ensemble. Le risque est ainsi réduit mais pas
complètement éliminé.
Question 8
Les investisseurs demanderont un taux de rendement plus élevés sur les investissements
présentant plus de risques en raison de leur aversion au risque. Pour qu'ils investissent,
certains individus demanderont une compensation supplémentaire sous forme d'un rendement
supérieur. Ceci n'implique pas qu'ils ont moins d'aversion au risque, seulement qu'ils sont
prêts à supporter ce risque en échange d'une compensation plus grande.
Exercice 1
a) E(rendement)= somme des rendements pondérée par leurs probabilités
= 0.1*100 + 0.5*50 + 0.7*10= 27$
b) Var(rendement)= somme des déviations p/r à la moyenne au carré
Var(r)= 0.1*(100-27)²+0.2*(50-27)²+0.7*(10-27)²=841$
c) Un individu neutre au risque paierait le rendement attendu de la loterie soit 27$.
245
1-803 Analyse Microéconomique
Solutions aux Exercices
Exercice 5
a) E(R)= 0.999 * -1 000 000 + 0.001 * 1 000 000 000 = 1 000 $
V(R)= 0.999 *(-1x10 - 1000)²+ 0.001 * (1x10 -1000)²
= 1.001x10
b) Puisque Sam est neutre au risque et que le résultat attendu est positif (1000$), Sam n'est
pas prêt acheter d'assurance à aucun prix positif.
c) L'entrée des Japonais réduit la probabilité d'un haut rendement pour Sam. Supposons une
baisse à 0.0001. Dans ce cas, le rendement attendu serait 0.9999* -1000000 + 0.0001 *
1000000000 = -899 900. Ainsi, vous pourriez hausser sa prime substantiellement mais
Sam, ne connaissant pas le comportement des Japonais, refusera vos offres visant à assurer
ses pertes.
Exercice 6
a) Natasha est risquophobe. Supposons qu'elle détienne 10 000$ et qu'on lui offre de parier 1
000$ avec 50% de chance de gain. Son utilité avec 10 000$ est 3.162 et son utilité
espérée est 0.5*9+0.5*11=3.158< 3.162. Elle refuserait donc de parier. Si elle était
neutre au risque, elle serait indifférente entre le 10000$ et le pari, si elle aimait le risque,
elle préférerait parier.
b) L'espérance de l'utilité du nouvel emploi = 0.5*5+0.5*16=3.118 ce qui est moins que
son utilité avec ses 10 000$=3.162.
c) Natasha serait prête à payer une prime de risque égale à la différence entre 10000$ et
l'utilité de son pari. EU=3.118 dans la fonction d'utilité 3.118=y donc le revenu associé au
pari est égal à 9722$. Natasha serait prête à payer une assurance égale à la prime de risque:
10 000 - 9 722= 278$.
246
1-803 Analyse Microéconomique
Solutions aux Exercices
Chapitre 6
Question 2
Tout facteur de production ajouté initialement au processus de production a une contribution positive
(produit marginal croissant). À un certain point, les unités supplémentaires de facteurs contribuent de
moins en moins à la production. Donc, le produit marginal décroît.
Question 4
On s'intéresse à la contribution du dernier employé à la production totale (PT), i.e. la production
marginale (Pm). Lorsque la production moyenne (PM) commence à décroître, c'est là où Pm=PM.
On devra arrêter d'engager des travailleurs seulement lorsque PT commence à décroître. Quand PM
commence à diminuer, la Pm du dernier employé < la PM des autres employés.
Question 7
La plupart des firmes ont des fonctions de production à rendement d'échelles croissant, constant et
décroissant. À un faible niveau de production, une hausse proportionnelle des facteurs implique une
hausse plus que proportionnelle dans la production. Au fur et à mesure que la firme croît, elle ne peut
accroître sa production au même rythme que ses facteurs.
Exercice 1
Voir solutionnaire dans le manuel.
Exercice 2
# de facteurs
variable
Production
totale
Prod. marginal des
fact. var.
Prod. moyen des
fact. var.
0
0
-
-
1
150
150
150
2
400
250
200
3
600
200
200
4
760
160
190
5
910
150
182
6
900
-10
150
247
1-803 Analyse Microéconomique
Solutions aux Exercices
Exercice 6 dans 2e édition et 7 dans 3e édition
a)
b)
On remplace K par l.K et L par l.L. Alors Q = 0,5.K.L  Q* = 0,5 (l.K).(l.L)
= (0,5.K.L).l2 = Q.l2 > l.Q  rendements d'échelle croissant.
Le rendement à l'échelle réfère la relation entre un accroissement proportionnel des
facteurs et la production, si représente un accroissement proportionnel des facteurs,
F( K, L) > F(K,L)  rend. croissants à l'échelle
F( K, L) = F(K,L)  rend. constants à l'échelle
F( K, L) < F(K,L)  rend. décroissants à l'échelle
De la même façon, Q = 2.K + 3.L
Þ Q* = 2.(l.K) + 3.(l.L) = (2.K + 3.L).l = Q.l  rend. constants à l'échelle
Exercice 7 dans 2e édition et 8 dans 3e édition:
a)
Notons Q1, la production de DISK et Q2, la production de FLOPPY et X, une
certaine quantité égale de K et de L pour les deux firmes. Alors,
(0.5+0.5)
Q1 = 10.X0.5 .X0.5 = 10.X
= 10.X et
0.6
0.4
(0.6+0.4)
Q2 = 10.X .X = 10.X
= 10.X
Puisque Q1 = Q2, les deux firmes produisent le même output avec la même quantité
de facteurs de K et L.
b)
Avec le facteur K fixe K=9, on a que:
Q1 = 30.L0.5 et Q2 = 37.37.L0.4 , pour déterminer la fonction de production ayant la
productivité marginale la plus élevée, examinons le tableau suivant:
L
Q1
Pm1
Q2
Pm2
0
0.0
-
0.0
-
1
30.00
30.00
37.37
37.37
2
42.43
12.43
49.31
11.94
3
51.96
9.53
57.99
8.68
4
60.00
8.04
65.06
7.07
Pour chaque unité de L au-dessus de 1 unité, la productivité marginale Pm de L est plus
élevée pour la firme 1, DISK Inc.
248
1-803 Analyse Microéconomique
Solutions aux Exercices
Exercice 8 dans 2e édition et 9 dans 3e édition:
a)
Pour le travail (L) fixe et le capital (K) variable;
K=4 
Q = 100.40.8 .490.2 = 660
K=5 
Q = 100.50.8 .490.2 = 789

K=6 
Q = 100.60.8 .490.2 = 913

0.8
0.2
K=7 
Q = 100.7 .49 = 1033

PmK = 129
PmK = 124
PmK = 120
Pour le capital (K) fixe et le travail (L) variable;
L = 49 
Q = 100.40.8 .490.2 = 660
L = 50 
Q = 100.40.8 .500.2 = 662.89 
PmL = 2.67
L = 51 
Q = 100.40.8 .510.2 = 665.52 
PmL = 2.63
0.8
0.2
L = 52 
Q = 100.4 .52 = 668.11 
PmL = 2.59
Notons que le produit marginal du capital et du travail est décroissant lorsque l'input
variable augmente.
b)
Q = 100.k0.8 .L0.2
Q* = 100.(l.k)0.8 .(l.L)0.2
= 100.k0.8 .L0.2 l(0.8+0.2)
= Q.l  rendements d'échelle constant
249
1-803 Analyse Microéconomique
Solutions aux Exercices
Chapitre 7
Question 1
Les coûts explicites sont ceux qui requièrent une transaction monétaire. Un coût implicite est
un coût économique qui implique l'utilisation de ressources. Dans le cas du comptable, celuici échange son temps contre de l'argent (i.e.: transaction monétaire). Par conséquent, il s'agit
d'un coût explicite.
Question 6
Si le coût variable moyen augmente (diminue), cela signifie que la dernière unité produite
accroît le coût variable total plus (moins) que ne l'a fait l'unité précédente, en moyenne. Dans
ce cas, le coût marginal est au-dessus (en-dessous) du coût variable moyen. Si le coût
marginal est au-dessus du coût variable moyen, le coût variable moyen augmente.
Exercice 1
a)
Le coût variable de produire une unité supplémentaire (coût marginal) est constant à
1 000 $, par conséquent, le coût variable moyen est constant à 1 000 $.
Le coût fixe moyen étant de 10 000/Q, le coût variable total est 1 000 + 10 000/Q.
b)
Étant donné que le coût moyen total diminue avec Q, la firme devrait choisir un
niveau de production très élevé.
Exercice 2
Le coût d'opportunité du temps d'un travailleur correspond à la valeur du temps que ce
travailleur ne consacre pas à une autre activité, ce qui comprend les activités personnelles ou
de loisir. Bien qu'on ne puisse pas mesurer monétairement la valeur du temps d'une personne
sans emploi, cette valeur n'est certainement pas nulle.
Exercice 7 (exercice 6 dans la 3e édition)
Ce consultant ne sera plus engagé! Augmenter l'achalandage, que ce soit aux heures de pointe
ou en-dehors des heures de pointe, diminue toujours les coûts moyens. Si l'achalandage chute
à 10, les coûts moyens par passager augmentent à 3$. De plus, pendant les heures de pointe,
les autobus sont déjà pleins. Comment pourraient-ils contenir plus de passagers? Il est
préférable d'encourager les passagers à voyager plus en-dehors des heures de pointe, ce qui
peut être fait en augmentant les tarifs de pointe.
250
1-803 Analyse Microéconomique
Solutions aux Exercices
Exercice 9
a)
Quand Q=0, C=190. Les coûts fixes s'élèvent donc à 1 900 000 $.
b)
À 100 000 unités, Q=10. Le coût variable est 53Q = (53)(10) = 530 (ou 5 300 000 $).
Le coût variable moyen est CVT/Q = 530 / 10 = 53.
c)
Lorsque le coût variable moyen est constant, comme c'est le cas ici, le coût marginal
est égal au coût variable moyen, c'est-à-dire 53.
d)
À Q = 10 (i.e. 100 000 unités), le coût fixe moyen est 190/10 = 19.
e)
Les coûts fixes passent de 190 à 195, les coûts variables de 53 à 48,5. Les coûts fixes
incluent également les frais d'intérêt: 31. L'équation de coûts devient donc:
C = 195 + 48,5Q + 3 I.
Chapitre 8
Question 1.
Les pertes surviennent quand les revenus ne couvrent pas les coûts. Les revenus peuvent être
plus grands que les coûts variables sans toutefois couvrir les coûts fixes. À court terme, la
firme peut continuer à produire quand les revenus sont supérieurs aux coûts variables. À long
terme, tous les coûts sont variables donc tous les coûts doivent être couverts si la firme veut
rester en affaires. À court terme, la firme peut encourir des pertes avant une hausse anticipée
des prix ou une baisse des coûts de production.
Question 3
L'hypothèse clé de la concurrence est qu'il n'y a pas de barrières à l'entrée pour les nouveaux
entrants dans l'industrie. Donc, tant que le profit est positif, il y aura des nouveaux entrants.
À long terme, le nombre élevé de nouvelles firmes affectera les prix à la baisse, et le profit
économique deviendra nul.
Question 5
Les firmes n'entrent dans une industrie que si elles espèrent un profit positif. Ces profits
positifs à court terme sont assez élevés pour encourager l'entrée sur le marché et ce, même si
le profit à long terme est nul.
Question 6
Si on suppose que il n'y a aucun obstacle à la concurrence, alors des rendements à l'échelle
croissants peuvent diminuer le nombre de firmes à long terme. Plus les firmes produisent,
plus leurs coûts diminuent avec des rendements d'échelle croissants. Les grosses firmes sont
capables de vendre à un prix plus bas, or les petites firmes sortent du marché à long terme et il
ne reste qu'un petit nombre de firmes.
251
1-803 Analyse Microéconomique
Solutions aux Exercices
Question 7
En désaccord, puisque si le prix du marché diminue, les firmes diminuent leur production. Si
le prix diminue en bas du coût total moyen, elles continuent de produire à court terme
seulement. Si le prix diminue en bas du coût variable moyen, elles s'arrêtent de produire à
court terme. Donc, la survie des firmes dépend en partie de l'ampleur de la baisse de prix.
Exercice 1. Voir solutionnaire à la fin du manuel
Exercice 2
Le tableau suivant montre les revenus, les coûts et les profits si les coûts fixes (CF) sont de
50 $, 100 $ ou 150 $.
Output
(units)
0
1
2
3
4
5
6
7
8
9
10
11
Price Revenue
($/unit)
($)
40
40
40
40
40
40
40
40
40
40
40
40
0
40
80
120
160
200
240
280
320
360
400
440
Total
Cost
FC=50
50
100
128
148
162
180
200
222
260
305
360
425
Profit Marginal Total
Profit
Total
Profit
($)
Cost
Cost
($)
Cost
($)
FC=50
FC=50 FC=100 FC=100 FC=150 FC=150
-50
-60
-48
-28
-2
20
40
58
60
55
40
15
50
28
20
14
18
20
22
38
45
55
65
100
150
178
198
212
230
250
272
310
355
410
475
-100
-110
-98
-78
-52
-30
-10
8
10
5
-10
-35
150
200
228
248
262
280
300
322
360
405
460
525
La firme maximise son profit lorsque CF=100 $, à 8 unités. Elle minimise ses pertes lorsque
CF=150 $ au même nombre d'unités. Les coûts fixes n'influencent pas la quantité optimale,
car ils n'influencent pas les coûts marginaux.
Exercice 3. Voir solutionnaire à la fin du manuel
Chapitre 10
Question 1
Lorsque le coût marginal est supérieur au revenu marginal, le coût associé à la dernière unité
produite est supérieur au revenu qu'elle rapporte. La firme augmenterait son profit en ne
produisant pas cette dernière unité. Elle devrait donc réduire sa production jusqu'au point où
Cm=Rm.
Question 2
On peut montrer que cette mesure du pouvoir de marché est égale à l'inverse de l'élasticitéprix de la demande. Ce qui veut dire que lorsque l'élasticité-prix de la demande augmente
(diminue), l'inverse de l'élasticité-prix diminue (augmente) et le pouvoir de marché de la
firme diminue (augmente). Ainsi, lorsque l'élasticité-prix augmente, la firme ne peut pas
augmenter son prix au-dessus du coût marginal autant qu'elle voudrait.
252
-150
-160
-148
-128
-102
-80
-60
-42
-40
-45
-60
-85
1-803 Analyse Microéconomique
Solutions aux Exercices
Question 4
L’intensité du pouvoir de marché que détient une firme dépend de l'élasticité-prix de la
demande à laquelle elle fait face. Ainsi, dès qu'une firme fait face à une demande dont
l'élasticité-prix est inférieure à l'infini (en valeur absolue), elle détient une forme de pouvoir
monopolistique. Ce pouvoir se manifeste lorsque la firme différencie son produit ou lorsqu'il
y a des barrières à l'entrée de l'industrie.
Question 6
Trois facteurs déterminent l'élasticité-prix de la demande à laquelle fait face une firme:
1.
L'élasticité-prix de la demande de marché qui dépend de la facilité de trouver un
substitut. Plus il y a de substituts, plus les consommateurs ont d'alternatives, plus
l'élasticité-prix de la demande à la firme est forte.
2.
Le nombre de firmes sur le marché qui dépend des barrières à l'entrée. Plus il y a de
firmes, plus les consommateurs ont d'alternatives, plus l'élasticité-prix de la demande
à la firme est grande.
3.
L'interaction des firmes sur le marché qui dépend de la manière dont les autres firmes
vont réagir à un changement de prix. Si toutes les firmes changent leur prix, les
consommateurs auront peu d'incitatifs à changer de fournisseur.
Question 7
Lorsque une firme profite de son pouvoir de monopole, elle augmente son prix au-dessus du
coût marginal et les consommateurs achètent moins à un prix plus élevé. Le surplus des
consommateurs, c'est-à-dire la différence entre le prix qu'ils sont prêts à payer et le prix du
marché pour chaque unité consommée, s'en trouve diminué. Une partie de ce surplus est
transféré au producteur, l'autre constitue une perte sèche pour la société puisque personne ne
s'en accapare. De plus, étant donné que les firmes produisent moins, elles perdent des
revenus sur les unités non produites.
Exercice 2
Étant un gros producteur d'équipements, Caterpillar Tractor a un pouvoir de marché et doit
donc considérer la courbe de demande quand il choisit les prix de ses produits. Étant leur
conseiller, vous devriez mettre l'accent sur la détermination de la demande de chaque produit.
Il y a trois facteurs importants:
i)
Quelles sont les similarités entre les produits offerts par Caterpillar et ceux offerts par
ses concurrents? Si ce sont des substituts proches, une petite augmentation des prix
de Caterpillar fera que les consommateurs iront acheter chez les compétiteurs.
ii)
Quel est l'âge du stock de tracteurs? Une augmentation de 5% des prix provoquera
une faible diminution de la demande de vieux tracteurs.
iii)
Les tracteurs étant un input capital dans la production agricole, quelle est la
profitabilité espérée du secteur agricole?
Si les revenus espérés des fermiers sont en train de diminuer, une augmentation du prix des
tracteurs provoquera une forte diminution de la demande.
Exercice 4 (exercice 3 dans la 3e édition)
253
1-803 Analyse Microéconomique
Solutions aux Exercices
Demande: P = 100 - 0.01Q
et
CT = 50Q + 30 000
a)
Maximiser les profits: Revenu marginal Rm = Coût marginal Cm
Équation de demande linéaire: P = a - bQ, où a = 100 et b = 0.01,
Revenu marginal Rm = a - 2bQ = 100 - 2(0.01)Q = 100 - 0.02Q.
Coût marginal Cm = dCT/dQ = 50.
Max profits: Rm = Cm  100 - 0.02Q = 50  Q = 2500 et
P = 100 - 00.1Q = 100 - 0.01(2500) = 75 ¢.
Profits = -30 000 + 50(2500) - 0.01(25002 ) = 32 500 soit 325 $ par semaine.
Exercice 5 (exercice 4 dans la 3e édition)
a)
Pour trouver la fonction de revenu marginal, on doit d'abord trouver la fonction de
demande: P = a-bQ où a = 27 est l'ordonnée à l'origine et -b = P/Q = -3/2, la pente.
Donc, P = 27 - 3/2Q.
Le revenu total est RT = PQ = 27Q - 3/2Q2 .
Le revenu marginal est Rm = 27 - 3Q.
b)
La quantité qui maximise le revenu du monopoleur est celle où Rm = Cm
27 - 3Q = 10  Q = 17/3 et P = 27 - 3/2*17/3 = 18,5 RT  PQ = 105.
CT = QCM. Or, Cm = constante  CM=Cm  CT = 10Q = 170/3.
 = RT - CT = 48 $.
c)
À l'équilibre, en situation de concurrence pure et parfaite, le prix serait égal au coût
marginal. On aurait P = 27 - 3/2Q = 10 et Q = 34/3.
Exercice 10 (exercice 9 dans la 3e édition)
a)
La quantité qui maximise le profit du monopoleur est celle où Rm = Cm.
Le coût marginal de court terme est SRMC = 5.
La demande est P = 100Q-½  RT = PQ = 100Q½ et Rm = 50Q-½ .
5 = 50Q-½  Q = 100 et P = 100 * 100-½ = 10.
= RT - CT = 10*100 - (2 000 + 5*100) = -1 500.
Bien que le profit soit négatif, la firme ne devrait pas arrêter la production car le prix
est supérieur au coût variable moyen. Si on arrêtait la production, la perte serait de
2 000.
b)
À long terme, le coût marginal est 6. On égalise Rm et Cm:
6 = 50Q-½  Q = 50/6 = 69,44 et P = 100*[(50/6)2 ]-½ = 12.
= RT - CT = 416,67.
La firme devrait poursuivre ses activités.
c)
À long terme, MMMT doit remplacer tous ses facteurs de production fixes. Par
conséquent, le coût marginal à long terme devrait être supérieur à celui à court terme.
Chapitre 12
254
1-803 Analyse Microéconomique
Solutions aux Exercices
Question 1
Les 2 principales caractéristiques d'un marché de concurrence monopolistique sont: 1) les
firmes compétitionnent en vendant des produits différenciés qui sont hautement mais pas
parfaitement substituables
2) il est facile d'entrer ou de sortir de ce marché. Quand une
nouvelle firme y entre, la courbe de demande de chacune des firmes existantes se déplace vers
la gauche, ce qui amène une réduction des parts de marché. Ce déplacement dépend des
préférences des consommateurs pour le produit du nouvel arrivant.
Question 2
La pente de la courbe de demande de la firme est fonction de l'élasticité-prix de la demande
du produit de cette firme. L'élasticité-prix de la demande pour une firme est plus grande que
l'élasticité-prix pour le marché car il est plus facile pour les consommateurs de changer pour
un produit hautement substituabable fabriqué par une autre firme que de changer la
consommation pour un produit complétement différent. En concurrence parfaite, la firme fait
face à une courbe de demande
parfaitement élastique tandis qu'en concurrence
monopolistique, même si elle est plus plate que la demande du marché, la demande est plus
raide que celle en concurrence permettant ainsi des profits positifs. Les profits dans le court
terme incitent d'autres firmes à entrer sur le marché mais à mesure que ceci se fait, la
demande des firmes existantes et leur revenu marginal se déplacent vers la gauche, réduisant
la quantité maximisant le profit. Éventuellement, les prix et les profits
tomberont,
ne
laissant aucun incitatif à l'entrée.
Question 3
Pour: Trop de marques différentes de n'importe quel produit est un signe de capacité
excédentaire, ce qui implique un niveau de production inférieur à celui minimisant les coûts
moyens.
Contre: Les consommateurs valorisent leur liberté de choisir parmi un éventail de produits
compétitifs. Cependant, il peut y avoir des effets nuisibles sur la concurrence quand il y a trop
de marques pour un produit très peu différencié.
Question 8
255
1-803 Analyse Microéconomique
Solutions aux Exercices
Parce que les firmes ne peuvent explicitement fixer un prix, elles le fixent de façon implicite.
L'une des collusions possibles est de suivre un leader dans le prix. Ce leader de prix, souvent
un dominant dans l'industrie, détermine le prix maximisant son profit en calculant la courbe
de demande à laquelle il fait face. Il soustrait la quantité offerte par chaque firmes à ce prix, ce
qui reste est sa demande. Associé avec cette demande est la courbe de revenu marginal. Les
firmes choisissent les quantités en égalant la demande au revenu marginal. Le prix du marché
est le prix auquel la quantité maximisant le profit du leader se vend sur le marché. À ce prix,
ceux qui le suivent offrent le reste du marché.
Question 9
Il existe deux conditions pour qu’un cartel soit efficace : 1) la demande doit être inélastique
au prix et 2) le cartel doit être en mesure de contrôler toute l’offre, sinon, il faut que l’offre
des non-membres soit inélastique. L’OPEC a réussi à court terme car la demande et l’offre à
court terme étaient inélastiques. Le CIPEC n’a pas réussi car la demande et l’offre des nonCIPEC étaient très élastiques. De plus, un cartel fait face à deux problèmes organisationnels :
1) la décision du prix et de la division du marché et 2) suivre les règlements.
Chapitre 11
Question 2
La courbe de demande à laquelle le vendeur d’automobile fait face est comprise entre le prix
suggéré par le détaillant plus l’augmentation du concessionnaire, et le prix du détaillant plus
les frais d’administration et d’inventaire. À travers un processus de marchandage, le vendeur
essaye de déterminer le prix de réserve de l’acheteur pour enfin fixer le prix de vente. Si le
prix fixé > prix de réserve, la vente n’a pas lieu ou le profit du vendeur est perdu.
Question 6
La discrimination par les prix implique la séparation des consommateurs dans différents
marchés, soit en fonction de certaines caractéristiques, soit géographiquement, soit en
fonction du temps. C'est ce qu'on fait dans ce cas-ci: on exige des prix différents aux
consommateurs à des heures différentes. On peut ainsi augmenter le surplus des
consommateurs en exigeant un prix plus faible aux consommateurs dont l'élasticité est plus
forte que celle de l'ensemble du marché. La plupart des compagnies de téléphone vont
procéder de la sorte et exiger un prix différent selon l'heure de la journée: pendant les heures
d'affaires, le soir, la nuit et les fins de semaine. Les gens ayant une élasticité-prix plus grande
vont téléphoner lors des périodes où les tarifs sont le plus près de leur prix de réserve.
Question 8
La vente d'un rasoir Gillette peut se faire sous la forme d'une tarification en deux parties si les
256
1-803 Analyse Microéconomique
Solutions aux Exercices
lames et le rasoir sont vendus séparément. Si Gillette n'a pas de pouvoir monopolistique sur
le marché des lames, il fixe son prix au coût marginal et ne peut pas s'accaparer du surplus des
consommateurs. Par contre, s'il détient un certain pouvoir de monopole, il a avantage à
choisir le prix qui maximise ses profits.
Exercice 1
Voir solutionnaire à la fin du manuel.
Exercice 2
Vrai. On suppose que le coût marginal de présenter le film est zéro, c'est-à-dire que les coûts
sont fixes et ne varient pas avec le nombre de voitures. On peut donc traiter cette question
comme un problème de tarification en deux parties: le ciné-parc fixe son prix d'entrée de
façon à capturer le surplus des conducteurs ("signe viewer") et charge un prix positif, le
second tarif, à chaque passager.
Exercice 3
Cette pratique est possible parce que les vendeurs d'automobiles peuvent appliquer une
discrimination par les prix étant donné que les individus intéressés par les options de luxe ont
une fonction de demande beaucoup plus inélastique que ceux intéressés uniquement par la
voiture et les options de base.
Exercice 4
a)
Pour des marchés séparés, BMW choisit le niveau de QE et QU qui maximise ses
profits. = TR - TC = (QE PE + QU PU ) = [(QE +QU ) * 15 + 20 000]
On trouve Pi dans les équations de demande et on l'utilise dans l'équation de profit:
 = QE * (45 - QE /40) + QU * (55 - QU /100) - [(QE + QU ) * 15 + 20 000].
On calcule les dérivées et on les pose égales à zéro:
/QE = 45 - QE /200 - 15 = 0 et p/QU = 55 - QE /50 - 15 = 0.
On trouve QE = 6 000, PE = 30 000, QU = 2 000 et PU = 35 000. En remplaçant dans
l'équation de profit, on obtient = 110 millions.
b)
Si BMW charge le même prix dans les deux marchés, on utilise Q = QE + QU dans
l'équation de profit et on trouve le prix optimal:
 = TR - TC = Q * P - 15Q + 20 000 = Q * (P - 15) + 20 000.
En remplaçant Q par QE + QU dans l'équation, = (23 500 - 500 * P) * (P - 15) + 20
000. On maximise en dérivant par rapport à P et en posant l'équation égale à 0:
/P = 23 500 + 7 500 - 1 000 * P = 0
Ainsi, P = 31 000, QE = 5 600 et QU = 2 400. Les profits s'élèvent à 108 millions de
dollars.
257
Téléchargement